LSAT and Law School Admissions Forum

Get expert LSAT preparation and law school admissions advice from PowerScore Test Preparation.

User avatar
 Dave Killoran
PowerScore Staff
  • PowerScore Staff
  • Posts: 5852
  • Joined: Mar 25, 2011
|
#85999
Complete Question Explanation
(The complete setup for this game can be found here: lsat/viewtopic.php?f=303&t=7341)

The correct answer choice is (D)

If O and P do not attend the same class, then P attends the class with H and either N or S. And, based on our prior analysis, P must attend the fifth or sixth class.

Answer choice (A) is incorrect because if G attends the fifth class, then P, H, and N or S attend the sixth class. This results in no possible class for O to attend.

Answer choice (B) is incorrect because, as mentioned above, H must attend the fifth or sixth classwith P.

Answer choice (C) is incorrect because if I attends the fourth class, then from the sequence G would have to attend the fifth class, which was shown impossible in answer choice (A).

Answer choice (D) is the correct answer.

Answer choice (E) is incorrect because if S attends the second class, then N must attend the first class, resulting in P not being able to attend the fifth or sixth class with two other students.
 studyhelp20
  • Posts: 28
  • Joined: Dec 09, 2020
|
#82414
Dear Power Score Staff,

Could you please provide an explanation of the correct answer for this question? Along with explanations of the incorrect answers? Thanks for the help.

Sincerely,
Brennan
 Rachael Wilkenfeld
PowerScore Staff
  • PowerScore Staff
  • Posts: 1358
  • Joined: Dec 15, 2011
|
#82513
Hi Brennan,

We can get a great local diagram here once we know that O and P are in separate classes. This gives us a long chain.

N/S :longline: K :longline: IL :longline: G :longline: (O, PH N/S)

N/S are always first as the only males who can go before K, and one of whom must go before K. K is next in the sequence. IL must go after K, as K is the first female student. G must go after IL. Finally, we know that O and the group with P are both after G. This uses all six slots. The only uncertainty in this set up is if N/S is first, as well as the order of the O and PH N/S group in 5 and 6.

Our question asks us what could be true, so we are looking for something possible. Usually this means that the answer choice will focus on those uncertainties described above, but it could be an answer that we know must be true as well.

Let's look at the answer choices.

Answer choice (A): G cannot be 5th because O and P, in different classes, must both be after her.

Answer choice (B): H cannot be 3rd, because we need H to fill the class with P in either slot 5 or 6.

Answer choice (C): I cannot be 4th, because there wouldn't be room for G, O and P to all be in separate classes after her.

Answer choice (D): N COULD be 5th here. If S is in first, N could be with P and H in slot 5 after G. This is our correct answer, and as expected, focuses on the uncertainty present in the chain.

Answer choice (E): S cannot attend the second class, because we only have 4 solo classes. The group classes are IL and P and two other students. if S is class 2, N would have to be class 1. K is a solo class, G is a solo class, and O is a solo class (in this local question at least. N, G, K, O, and S would be 5 solo classes, which is too many to have here.

Hope that helps
Rachael

Get the most out of your LSAT Prep Plus subscription.

Analyze and track your performance with our Testing and Analytics Package.